Diễn Đàn MathScopeDiễn Đàn MathScope
  Diễn Đàn MathScope
Ghi Danh Hỏi/Ðáp Thành Viên Social Groups Lịch Ðánh Dấu Ðã Ðọc

Go Back   Diễn Đàn MathScope > Sơ Cấp > Đại Số và Lượng Giác > Các Bài Toán Đã Được Giải

News & Announcements

Ngoài một số quy định đã được nêu trong phần Quy định của Ghi Danh , mọi người tranh thủ bỏ ra 5 phút để đọc thêm một số Quy định sau để khỏi bị treo nick ở MathScope nhé !

* Nội quy MathScope.Org

* Một số quy định chung !

* Quy định về việc viết bài trong diễn đàn MathScope

* Nếu bạn muốn gia nhập đội ngũ BQT thì vui lòng tham gia tại đây

* Những câu hỏi thường gặp

* Về việc viết bài trong Box Đại học và Sau đại học


Ðề tài đã khoá Gởi Ðề Tài Mới
 
Ðiều Chỉnh Xếp Bài
Old 09-07-2011, 08:19 AM   #1636
leviethai
+Thành Viên+
 
Tham gia ngày: Nov 2008
Đến từ: Thành phố Hồ Chí Minh. Nhưng quê tôi là Ninh Bình.
Bài gởi: 513
Thanks: 121
Thanked 787 Times in 349 Posts
Gửi tin nhắn qua Yahoo chát tới leviethai
Anh Huyện à, hình như điều này không đúng

Trích:
$\[c \ge \left| {c - a} \right|.\] $

[RIGHT][I][B]Nguồn: MathScope.ORG[/B][/I][/RIGHT]
 
leviethai is offline  
Old 09-07-2011, 08:38 AM   #1637
Nguyenhuyen_AG
+Thành Viên+
 
Nguyenhuyen_AG's Avatar
 
Tham gia ngày: Apr 2010
Bài gởi: 300
Thanks: 35
Thanked 307 Times in 151 Posts
Trích:
Nguyên văn bởi leviethai View Post
Anh Huyện à, hình như điều này không đúng
Sao kỳ vậy ta . Chắc sáng sớm còn mơ ngủ chưa tỉnh táo, để anh tìm cách chữa cháy lời giải.
[RIGHT][I][B]Nguồn: MathScope.ORG[/B][/I][/RIGHT]
 
__________________
Nguyen Van Huyen
Ho Chi Minh City University of Transport
Nguyenhuyen_AG is offline  
Old 09-07-2011, 08:55 AM   #1638
leviethai
+Thành Viên+
 
Tham gia ngày: Nov 2008
Đến từ: Thành phố Hồ Chí Minh. Nhưng quê tôi là Ninh Bình.
Bài gởi: 513
Thanks: 121
Thanked 787 Times in 349 Posts
Gửi tin nhắn qua Yahoo chát tới leviethai
Trích:
Nguyên văn bởi codinh24 View Post
Chứng minh bất đẳng thức sau:
với $a,b,c $ là các số thực dương.
Bất đẳng thức này tương đương với bất đẳng thức sau

Trích:
Cho $a,b,c $ là các số thực dương thỏa mãn $xyz=1 $. Chứng minh rằng
$\[\frac{1}{{\sqrt {4{x^2} + x + 4} }} + \frac{1}{{\sqrt {4{y^2} + y + 4} }} + \frac{1}{{\sqrt {4{z^2} + z + 4} }} \le 1.\] $
Bài này có cách giải khá thú vị. Ta có bất đẳng thức sau

$\[\frac{1}{{\sqrt {4{x^2} + x + 4} }} \le \frac{1}{2} \cdot \frac{{x + 1}}{{{x^2} + x + 1}}.\] $

Như vậy, ta có
$\[\sum {\frac{1}{{\sqrt {4{x^2} + x + 4} }}} \le \frac{1}{2}\sum {\frac{{x + 1}}{{{x^2} + x + 1}}} \] $

Như vậy ta cần chứng minh
$\[\sum {\frac{{x + 1}}{{{x^2} + x + 1}}} \le 2\] $

Tương đương với
$\[\sum {\frac{{{x^2}}}{{{x^2} + x + 1}}} \ge 1.\] $

Đây là một bất đẳng thức quen thuộc và có thể chứng minh bằng Cauchy Schwarz. Ta có điều phải chứng minh. $\hfill \Box $

[RIGHT][I][B]Nguồn: MathScope.ORG[/B][/I][/RIGHT]
 
leviethai is offline  
The Following 2 Users Say Thank You to leviethai For This Useful Post:
Mệnh Thiên Tử (09-07-2011), Mr_Trang (10-07-2011)
Old 09-07-2011, 09:57 AM   #1639
birain9x
+Thành Viên+
 
birain9x's Avatar
 
Tham gia ngày: Feb 2011
Bài gởi: 119
Thanks: 28
Thanked 41 Times in 23 Posts
Cho $a,b,c>0 $ có tổng là 3. Chứng minh rằng
$\sqrt{\frac{a}{b^2+3}}+\sqrt{\frac{b}{c^2+3}}+ \sqrt{\frac{c}{a^2+3}}\le\frac{3}{2} $

@xin lỗi mọi người,hôm qua em gõ sai bài này.Cám ơn anh Huyện ạ.
[RIGHT][I][B]Nguồn: MathScope.ORG[/B][/I][/RIGHT]
 

thay đổi nội dung bởi: birain9x, 09-07-2011 lúc 09:59 AM
birain9x is offline  
Old 09-07-2011, 10:25 AM   #1640
daiduong1095
+Thành Viên+
 
daiduong1095's Avatar
 
Tham gia ngày: Sep 2010
Đến từ: CVP-Math
Bài gởi: 287
Thanks: 13
Thanked 210 Times in 112 Posts
Gửi tin nhắn qua Yahoo chát tới daiduong1095
Cho $a,b,c>0 $.Cmr:
$(a^3+b^3+c^3)^2 \ge(a^4+b^4+c^4)(ab+bc+ca) $
[RIGHT][I][B]Nguồn: MathScope.ORG[/B][/I][/RIGHT]
 
__________________
daiduong1095 is offline  
Old 09-07-2011, 10:42 AM   #1641
daiduong1095
+Thành Viên+
 
daiduong1095's Avatar
 
Tham gia ngày: Sep 2010
Đến từ: CVP-Math
Bài gởi: 287
Thanks: 13
Thanked 210 Times in 112 Posts
Gửi tin nhắn qua Yahoo chát tới daiduong1095
Trích:
Nguyên văn bởi ruang0 View Post
Có $(a+b+c)^2 \ge 3(ab+bc+ca) $
$\Rightarrow ab+bc+ac \le \frac{(a+b+c)^2}{3} = 3 $
$\Rightarrow \frac{a}{\sqrt{a^2+3}} \le \frac{a}{\sqrt{a^2+ab+bc+ca}} $
$ =\frac{a}{\sqrt{(a+b)(a+c)}} $

$\le \frac{1}{2}(\frac{a}{a+b}+\frac{a}{c+a}) $Chứng minh tương tự
Và cộng từng vế của 3 bất đẳng thức lại ta có đpcm.
Lời giải sai rồi bạn ạ.Trên tử sao bạn lại cho "a" ra ngoài dấu căn mà bạn làm đề bài khác hoàn toàn rồi.Mỗi $ab+bc+ca \le3 $ là đúng.Còn lại nhầm hết cả!!!
[RIGHT][I][B]Nguồn: MathScope.ORG[/B][/I][/RIGHT]
 
__________________
daiduong1095 is offline  
The Following User Says Thank You to daiduong1095 For This Useful Post:
je.triste (09-07-2011)
Old 09-07-2011, 02:42 PM   #1642
hien123
+Thành Viên+
 
Tham gia ngày: Sep 2010
Đến từ: THPT chuyên Phan Bội Châu, Nghệ An
Bài gởi: 353
Thanks: 19
Thanked 261 Times in 165 Posts
Cho a, b, c là các số thực dương thỏa mãn abc=1. Chứng minh rằng:

$\sum \frac{1}{\sqrt{2a^{2}+6a+1}}\geq 1 $
[RIGHT][I][B]Nguồn: MathScope.ORG[/B][/I][/RIGHT]
 
__________________
$z=\left | z \right |e^{i\varphi } $
hien123 is offline  
Old 09-07-2011, 03:20 PM   #1643
hien123
+Thành Viên+
 
Tham gia ngày: Sep 2010
Đến từ: THPT chuyên Phan Bội Châu, Nghệ An
Bài gởi: 353
Thanks: 19
Thanked 261 Times in 165 Posts
Trích:
Nguyên văn bởi birain9x View Post
Do $abc=1 $ nên tồn tại các số $x,y,z>0 $ mà $a=\frac{yz}{x^2},b=\frac{xz}{y^2},c=\frac{xy}{z^2} $
Ta cần chứng minh bdt $\sum \frac{x^2}{\sqrt{2y^2z^2+6yzx^2+x^4}}\geq 1 $
Ta có $VT^2.(\sum (x^2(2y^2z^2+6yzx^2+x^4)))\geq (x^2+y^2+z^2)^3 $ (bdt hóc đờ)
Mà $(x^2+y^2+z^2)^3-\sum (x^2(2y^2z^2+6yzx^2+x^4))=3\sum x^4(y-z)^2\geq 0 $.Suy ra dpcm.
Lời giải đơn giản hơn:
Đặt$ a=x^{9}, b=y^{9}, c=z^{9} $, hãy chứng minh rằng:
$\frac{1}{\sqrt{2x^{18}+6x^{9}+1}}\geq \frac{1}{x^{10}+x^{5}+1} $
Suy ra:
$\sum \frac{1}{\sqrt{2a^{2}+6a+1}}\geq\sum \frac{1}{x^{10}+x^{5}+1} $
Với chú ý bổ đề quen thuộc:
$\sum \frac{1}{m^{2}+m+1}\geq 1 $với mnp=1
Ta có đpcm
[RIGHT][I][B]Nguồn: MathScope.ORG[/B][/I][/RIGHT]
 
__________________
$z=\left | z \right |e^{i\varphi } $
hien123 is offline  
The Following User Says Thank You to hien123 For This Useful Post:
ilovehien95 (09-07-2011)
Old 09-07-2011, 03:28 PM   #1644
leviethai
+Thành Viên+
 
Tham gia ngày: Nov 2008
Đến từ: Thành phố Hồ Chí Minh. Nhưng quê tôi là Ninh Bình.
Bài gởi: 513
Thanks: 121
Thanked 787 Times in 349 Posts
Gửi tin nhắn qua Yahoo chát tới leviethai
Trích:
Nguyên văn bởi birain9x View Post
------------------------------

Bài toán này có rất nhiều cách giải.Cách giải sau đây rất nhẹ nhàng.
W.L.O.G,giả sử $a\geq b\geq c\Rightarrow a^4\geq b^4\geq c^4,bc\leq ac\leq ab $
Áp dụng bdt Trê bư sép thì $(a^4+b^4+c^4)(ab+bc+ca)\leq 3(a^4.bc+b^4.ac+c^4.ab)=3abc(a^3+b^3+c^3)\leq (a^3+b^3+c^3)^2 $
Vậy ta có dpcm.
"Nhẹ nhàng" quá nên chưa đúng bạn à, bạn coi lại bất đẳng thức Chebychev áp dụng thế nào nhé.
[RIGHT][I][B]Nguồn: MathScope.ORG[/B][/I][/RIGHT]
 
leviethai is offline  
Old 09-07-2011, 03:46 PM   #1645
birain9x
+Thành Viên+
 
birain9x's Avatar
 
Tham gia ngày: Feb 2011
Bài gởi: 119
Thanks: 28
Thanked 41 Times in 23 Posts
Trích:
Nguyên văn bởi leviethai View Post
"Nhẹ nhàng" quá nên chưa đúng bạn à, bạn coi lại bất đẳng thức Chebychev áp dụng thế nào nhé.
Cám ơn bạn.mình giải vội quá nên nhầm.
Mình giải lại vậy.
Ta có đẳng thức
$ (a^3+b^3+c^3)^2-(a^4+b^4+c^4)(ab+bc+ca)=\frac{1}{2}\sum ((a^2-b^2)^2+c^4)(a-b)^2\geq 0 $
[RIGHT][I][B]Nguồn: MathScope.ORG[/B][/I][/RIGHT]
 
birain9x is offline  
Old 09-07-2011, 05:06 PM   #1646
hien123
+Thành Viên+
 
Tham gia ngày: Sep 2010
Đến từ: THPT chuyên Phan Bội Châu, Nghệ An
Bài gởi: 353
Thanks: 19
Thanked 261 Times in 165 Posts
Trích:
Nguyên văn bởi daiduong1095 View Post
Cho $a,b,c>0 $.Cmr:
$(a^3+b^3+c^3)^2 \ge(a^4+b^4+c^4)(ab+bc+ca) $
Đổi biến pqr và chuẩn hóa p=1,ta được:
$9r^{2}+2r\left ( 3-11q \right )+1-2q^{3}+13q^{2}-7q\geq 0 $
Nếu $q\leq \frac{3}{11} $ BĐT hiển nhiên đúng
Nếu $q\geq \frac{3}{11} $ ta có:
$f'\left ( r \right )=2\left ( 9r+3-11q \right ) $
Xét q$\geq \frac{10}{33} $ thì $f'\left ( r \right )\leq 0 $$\Rightarrow $ $f\left ( r \right ) $ nghịch biến $\Rightarrow f(r)\geq f(\frac{1}{27}) $ Từ đó bằng cách khảo sát hàm theo q ta dễ có đpcm
Nếu $\frac{3}{11}\leq q\leq \frac{10}{33}\Rightarrow f'(r)=0\Leftrightarrow r=\frac{11q-3}{9} $. Từ đó bằng cách lập bảng biến thiên ta có: $f(r)\geq f\left ( \frac{11q-3}{9} \right ) $ Đến đây lại khảo sát hàm theo q ta dễ có đpcm
[RIGHT][I][B]Nguồn: MathScope.ORG[/B][/I][/RIGHT]
 
__________________
$z=\left | z \right |e^{i\varphi } $

thay đổi nội dung bởi: hien123, 09-07-2011 lúc 07:34 PM
hien123 is offline  
Old 09-07-2011, 10:22 PM   #1647
DaiToan
+Thành Viên+
 
Tham gia ngày: Oct 2010
Đến từ: THPT Chuyên Vĩnh Phúc
Bài gởi: 280
Thanks: 29
Thanked 361 Times in 123 Posts
Cho $a,b,c,d $ dương. Chứng minh rằng:
$4.\sqrt[{16}]{{\dfrac{{32a(a + b)(a + b + c)}}{{3(a + b + c + d)^3 }}}} + \sqrt[4]{{\dfrac{{24bcd}}{{(a + b)(a + b + c)(a + b + c + d)}}}} \le 5 $

[RIGHT][I][B]Nguồn: MathScope.ORG[/B][/I][/RIGHT]
 

thay đổi nội dung bởi: novae, 09-07-2011 lúc 10:24 PM
DaiToan is offline  
Old 09-07-2011, 10:53 PM   #1648
11112222
+Thành Viên+
 
11112222's Avatar
 
Tham gia ngày: Oct 2010
Đến từ: Địch Nhân Kiệt' house
Bài gởi: 55
Thanks: 15
Thanked 10 Times in 9 Posts
Bất đẳng thức với x,y,z thuộc R

Cho $x^2+y^2+z^2=2 $.
Chứng minh : $x+y+z\le2+xyz $
[RIGHT][I][B]Nguồn: MathScope.ORG[/B][/I][/RIGHT]
 
11112222 is offline  
Old 09-07-2011, 10:59 PM   #1649
Lan Phuog
+Thành Viên Danh Dự+
 
Lan Phuog's Avatar
 
Tham gia ngày: Mar 2010
Đến từ: Thái Bình
Bài gởi: 564
Thanks: 289
Thanked 326 Times in 182 Posts
Cho $x,y,z>0,xyz=1 $. Cmr: $\sum \frac{x^3}{z^3}(\frac{y^2}{x^2}+\frac{z^2}{y^2})^2 \ge \sum (\frac{x^4}{y}+\frac{x}{y^4})+2\sum\frac{x^2}{y^2} $
[RIGHT][I][B]Nguồn: MathScope.ORG[/B][/I][/RIGHT]
 
Lan Phuog is offline  
Old 09-07-2011, 11:07 PM   #1650
daiduong1095
+Thành Viên+
 
daiduong1095's Avatar
 
Tham gia ngày: Sep 2010
Đến từ: CVP-Math
Bài gởi: 287
Thanks: 13
Thanked 210 Times in 112 Posts
Gửi tin nhắn qua Yahoo chát tới daiduong1095
Trích:
Nguyên văn bởi 11112222 View Post
Cho $x^2+y^2+z^2=2 $.
Chứng minh : $x+y+z\le2+xyz $
Bạn tham khảo lời giải tại đây [Only registered and activated users can see links. ]
[RIGHT][I][B]Nguồn: MathScope.ORG[/B][/I][/RIGHT]
 
__________________
daiduong1095 is offline  
Ðề tài đã khoá Gởi Ðề Tài Mới

Bookmarks

Tags
bất đẳng thức

Ðiều Chỉnh
Xếp Bài

Quuyền Hạn Của Bạn
You may not post new threads
You may not post replies
You may not post attachments
You may not edit your posts

BB code is Mở
Smilies đang Mở
[IMG] đang Mở
HTML đang Tắt

Chuyển đến


Múi giờ GMT. Hiện tại là 01:35 AM.


Powered by: vBulletin Copyright ©2000-2024, Jelsoft Enterprises Ltd.
Inactive Reminders By mathscope.org
[page compression: 101.62 k/117.41 k (13.45%)]